Sei sulla pagina 1di 11

Exam FM/2 Practice Exam 2

Copyright c 2013 Actuarial Investment. 1. $500 is deposited into an account earning interest at a force of interest of interest is earned in the fourth year? (A) 450 (B) 500 (C) 2000 (D) 2500 (E) 3125 2. With all else held constant, which of the following are true regarding convexity? (I) Convexity of a portfolio with larger cashows is lower than convexity of a portfolio with smaller cashows. (II) Convexity calculated at a high interest rate is lower than convexity calculated at a lower interest rate. (III) Convexity of a single cashow later into the future is higher than convexity of a single cashow sooner. (IV) Convexity of a portfolio with higher modied duration is higher than convexity of a portfolio with lower modied duration. (A) (III) only (B) (II) and (IV) (C) (I), (II), and (III) (D) (II), (III), and (IV) (E) The answer is not given by any of (A), (B), (C), or (D). 3. Two bonds with annual coupons have the same yield rate j . The rst bond is a 10-year bond with face amount 1000 and a coupon rate q that is 2 percentage points higher than the bonds yield rate j . The second bond is a 10-year bond with face amount 1000 and a coupon rate r that is 3 percentage points lower than the bonds yield rate j . The rst bond is worth 350 more than the second bond. Calculate the price of the rst bond. (A) 870 (B) 981 (C) 1046 (D) 1140 (E) 1174 1
1 . 1+t

How much

4. Chris buys a call option with maturity in n months, a strike price of 23, and a premium of 3. His prot at maturity is 1.25. At the time he buys the call option, the price of the underlying asset is 25. After he buys the call option, the value of the underlying asset increases by 1% per month forever. The annual effective rate of interest is 4%. Calculate n. (A) 5 (B) 6 (C) 7 (D) 8 (E) 9 5. A loan makes level monthly payments of P at the end of each month for 36 months. The interest paid in the 6th month is 13. The annual effective interest rate is 2%. Calculate P . (A) 240 (B) 261 (C) 269 (D) 275 (E) 283 6. The price of a two-year zero-coupon bond with face amount 1000 is 905. Six-month forward rates are given in the table below. T (years) .5 1 1.5 2 i(T .5, T ) 2.6 2.5 X 2.6

The price of a two-year bond with face amount 1000 and coupons at a rate of r% convertible semiannually is 1030. Calculate r. (A) 3.3 (B) 5.1 (C) 6.7 (D) 7.8 (E) 8.8

7. An investor believes that a stock is very volitile and wants a position to capitalize on this belief. In which of the following positions is he most likely to invest? (A) Long position in a bear spread (B) Long position in a buttery spread (C) Long position in a straddle (D) Short position in a put ratio spread (E) Short position in a strangle 8. A company has a liability of 1000 in 4 years. The company wants to provide Redington immunization for the liability by purchasing two zero-coupon bonds are available for purchase at any face amount. Bond X matures in 2 years and bond Y matures in 5 years. The annual effective rate of interest is 5%. What amount of bond Y should be purchased to provide Redington immunization for this liability? (A) 400 (B) 500 (C) 600 (D) 700 (E) 800 9. Two 12-year loans with the same interest rate have annual payments at the end of each year. The combined interest paid in the 5th payments of the loans is 200. The rst loan is worth 1000 and has payments of 140. The other loan is worth X . Calculate X . (A) 1817 (B) 1857 (C) 1876 (D) 1906 (E) 1939 10. An annuity immediate with 30 annual payments is worth 392. In years that are divisible by n, the payment is 27. In all other years, the payment is 19. The annual effective rate of interest is 3%. Calculate n. (A) 5 (B) 6 (C) 7 (D) 8 (E) 9 3

11. The price of a stock is 85. The stock just paid a dividend of 8, and future dividends are expected to increase by 3% per year, beginning with the dividend to be paid one year from now. Calculate the yield rate i% earned from purchasing this stock. (A) 11.94 (B) 12.41 (C) 12.69 (D) 13.05 (E) 18.20 12. A company has liabilities of 1000 due in 6 years and 1400 due in 10 years. The company has an asset of 2390 available in 8 years. The annual effective rate of interest is i%. For what range of i are the companys liabilities immunized? (A) 0 i 2.8 (B) 1.4 i 16.7 (C) 2.8 i 36.2 (D) i 1.4, i 16.7 (E) i 2.8, i 36.2 13. A loan of 10,000 has n monthly payments beginning in one month. The principal paid in the 12th payment is 455.05, and the principal paid in the 21st payment is 465.25. Calculate n. (A) 22 (B) 23 (C) 24 (D) 25 (E) 26 14. A perpetuity due makes annual payments. The rst payment is 5.66 and payments increase by r% each year. The effective annual rate of interest is 10%. The perpetuity is worth 155.65. Calculate r. (A) 5.8 (B) 6.0 (C) 6.2 (D) 6.4 (E) 6.6 4

15. An investor purchases a portfolio that produces cashows of 200 at time 4, X at time 5, and 320 at time 8. It is known that the cashow of X at time 5 makes up 20% of the value of the portfolio and that the rate of interest per time period is 5%. Calculate the Macaulay duration of the portfolio. (A) 5.1 (B) 5.5 (C) 6.0 (D) 6.9 (E) 7.3 16. An investor believes that over the next several years, the value of an asset will have a large overall decrease in value, but that the value will experience signicant uctuation while it declines. Into which of the following contracts is the investor most likely to enter? (A) Long position in an American put option (B) Long position in a European call option (C) Short position in an American call option (D) Short position in a European call option (E) Short position in a European put option 17. The annual effective rate of interest is i. The present value of an annuity immediate with n annual payments of 1 is X . The present value of an annuity immediate with 2n annual payments of 1 is 1.5X . Find the accumulated value of 1 in n years. (A) 0.50 (B) 1.50 (C) 1.67 (D) 1.75 (E) 2.00

18. Bond X is a 10-year bond with annual coupons of 10%. For the forseeable future, it is available to purchase at par in any face amount. Molly buys $10,000 of bond X . As she recieves coupons from her initial purchase, she reinvests them by buying more of bond X . She then reinvests those coupon payments (all coupon payments not from the initial purchase) into an account paying annual interest at a rate of 6%. How much money (excluding the value of bonds not yet redeemed) does Molly have after she receives the face amount of the original $10,000 bond at the end of 10 years? (A) 15301 (B) 16619 (C) 18793 (D) 20210 (E) 23301 19. A stock is selling today for 25.00, and the three-month forward price for the stock is 25.50. The stock does not pay dividends. The annual effective rate of interest is 7.4%. An investor borrows 100,000 to exploit this arbitrage opportunity. What is the investors gain at the end of three months? (A) 200 (B) 400 (C) 500 (D) 700 (E) 900 20. An investor buys two puts for the same underlying asset that each mature in six months. Put X has a strike price of 48 and a premium of 3. Put Y has a strike price of 51 and a premium of 4. At maturity, the investor exercises both options and makes a net prot of 0. The annual effective interest rate is 5%. What is the underlying assets price at maturity? (A) 45.91 (B) 46.22 (C) 47.60 (D) 52.17 (E) 53.09

21. A perpetuity immediate makes annual payments. The rst payment is 1000. Subsequent payments increase by 100 until they reach 2000. All remaining payments are 2000. The annual effective rate of interest is 13%. Calculate the present value of the perpetuity. (A) 11866 (B) 12993 (C) 15385 (D) 16932 (E) 18903 22. A 12-year bond has an annual effective yield rate of 4% and makes semiannual coupons of X . It has a face amount of 1000 and a price of 1145.89. A 5-year bond has an annual effective yield rate of 4% and makes semiannual coupons of X 15. It has a face amount of 1000 and a price of P . Calculate P . (A) 822.67 (B) 861.55 (C) 901.83 (D) 934.32 (E) 1002.43 23. Which of the following are true regarding swaps? (I) An interest rate swap can be used to borrow at a lower interest rate than would otherwise be possible (II) An interest rate swap can be used to reduce uncertainty caused by changes in interest rates (III) In a swap, the present value of the remaining payments of each of the two sets of payments to be exchanged must remain equal for the term of the swap (A) (I) only (B) (II) only (C) (I) and (II) (D) (II) and (III) (E) The answer is not given by any of (A), (B), (C), or (D)

24. On January 1, a fund has 900. At some time during the year, a deposit is made of X . On December 31, the funds balance is 1500. , but if the deposit had been made one The dollar-weighted rate of return of the fund is 1 7 4 . month later, the dollar-weighted rate of return would have been 27 Calculate X . (A) 367 (B) 400 (C) 417 (D) 450 (E) 480 25. It is known that K1 < K2 < K3 < K4 < K5 . An investor purchases a portfolio that consists of long positions in a K1 -K2 -K3 buttery spread and a K4 -K5 2:1 call ratio spread. Over what interval(s) is the investors payoff positive? (A) (0, K1 ), (K3 , K5 ) (B) (K1 , K3 ), (K4 , K5 ) (C) (K1 , K3 ), (K4 , 2K5 K4 ) (D) (K1 , K3 K2 ), (K4 , K5 ) (E) The answer is not given by any of (A), (B), (C), or (D). 26. A 12-year bond has annual coupons of 5%. The amount for accumulation of discount in the 8th year is twice the amount for accumulation of discount in the 4th year. The price of the bond is equal to its face amount. Calculate the yield rate. (A) 4.2% (B) 5.0% (C) 5.4% (D) 5.9% (E) 6.3%

27. A 10-year loan makes 9 annual payments of 300 at the end of each year followed by a balloon payment of 700 at the end of year 10. After 3 years, the outstanding balance is 2000. The loan amount is X . Calculate X . (A) 2238 (B) 2382 (C) 2467 (D) 2529 (E) 2709 28. The annual effective rate of interest is 2.8%. The future value of a 24-year annuity due with annual payments of P is X . The present value of a 24-year annuity immediate with annual payments of P 2 is X . Calculate X . (A) 57 (B) 61 (C) 65 (D) 69 (E) 73 29. An investor purchases a futures contract to buy 10,000 shares of Protable, Inc. in 3 months at a price of X per share. There is a 4% commission on the futures contract. In 3 months, the price of one share of Protable, Inc. is 35.50. The investors net prot is 580. Calculate X. (A) 34.02 (B) 34.08 (C) 34.19 (D) 36.92 (E) 37.80 30. The rst payment of an annuity immediate with annual payments is 100 and subsequent payments increase by 2%. The nal payment is 200. If the annual effective rate of interest is 5%, what is the present value of the annuity? (A) 1997 (B) 2125 (C) 2159 (D) 3079 (E) 3187 9

31. Fund A, Fund B, Fund C, and Fund D each have a beginning balance of 1000 on January 1 and an ending balance of 1000 on December 31. Deposits and withdrawals are made according to the following schedule, with positive numbers representing deposits and negative numbers representing withdrawals. Date January 1 February 1 March 1 April 1 May 1 June 1 July 1 August 1 September 1 October 1 November 1 December 1 Fund A 50 100 50 100 50 100 50 100 50 100 Fund B Fund C 200 75 75 75 200 200 75 200 75 75 200 200 Fund D

Which of the funds has the highest dollar-weighted rate of return? (A) Fund A (B) Fund B (C) Fund C (D) Fund D (E) Two or more of the funds have the same dollar-weighted rate of return 32. All of the following options have the same maturity date and are for the same underlying asset. Given the right conditions, which of the following positions could offer an opportunity for arbitrage? (I) A long position in the asset, a sale of a zero-coupon bond, and a short position in a forward contract (II) A short position in a call option and a long position in a put option (III) A short position in a forward contract, a sale of a zero-coupon bond, and a long position in a convertible bond (A) (I) only (B) (I) and (II) (C) (II) and (III) (D) (I), (II), and (III) (E) The answer is not given by any of (A), (B), (C), or (D) 10

33. An investor writes a 4:1 put ratio with a net premium of 24. The maximum amount the investor could lose is 15, which occurs if the price of the underlying asset at expiration is 35. What is the maximum amount the investor could gain? (A) 24 (B) 90 (C) 125 (D) The investors maximum gain is unlimited. (E) There is not enough information given to determine the investors maximum gain. 34. An investor purchases a 20-year bond with annual coupons of 5%. The bond is callable immediately after any coupon payment in the 12th year or later. It has a face value of 1000 and a price of 927. The investor will earn an annual effective yield of at least j %. Calculate j. (A) 5.00 (B) 5.26 (C) 5.62 (D) 5.71 (E) 5.86 35. A loan is repaid with n level annual payments starting 1 year after the loan is taken out. The is two-thirds of the loan amount. The annual effective outstanding balance after payment n 2 rate of interest is 8%. Calculate n. (A) 16 (B) 18 (C) 20 (D) 22 (E) 24

11

Potrebbero piacerti anche